Calculating the Limit Using Riemann Sum with Starred Part?

Click For Summary
The discussion focuses on calculating a limit using Riemann sums, specifically addressing confusion around a starred part of the problem. Participants emphasize the importance of clearly presenting mathematical expressions, suggesting that the original poster should use LaTeX for clarity. There is a debate regarding the use of logarithmic functions, questioning whether to use log or ln consistently. Additionally, the need for careful step-by-step verification in calculations is highlighted, as misunderstandings can arise from miswritten expressions. Overall, the thread underscores the significance of clarity and precision in mathematical problem-solving.
devinaxxx

Homework Statement


http://i66.tinypic.com/aesd1u.png

can someone explain to me how can i get the limit using riemann sum especially the starred part? i was so confused thanks!

Homework Equations



The Attempt at a Solution


attempt at a solution in the picture
 
Physics news on Phys.org
I don't see a picture and I don't think this provider is appropriate. You could either load up your image instead, or far, far better, type in your problem, preferably using LaTex:
https://www.physicsforums.com/help/latexhelp/
 
aesd1u.png


Hello devina,

As you may know, PF keeps threads for years. You sure this tinypic link is sustainable :smile: ?

Any substantial difference between ##\log## and ##\ln## ? If not, then why not stick to one of the two ?

The stars are dazzling in front of my eyes. But even before the first two I lose track. What is at the left hand side of the first ##=## sign ? Surely not the limit and also not the expression. Nor the logarithm of the expression, if I read carefully. Help !
 

Homework Statement


Shouldn't it be something like
Calculate the limit $$\lim_{n\rightarrow\infty} {1\over n} \left ( (2n)!\over n! \right)^{1\over n} $$ using a Riemann sum ?
This because I have seldom seen an excercise saying ' calculate this sum of limit ' :rolleyes:

Homework Equations


Something like ##\int = \lim \sum ...## ??

The Attempt at a Solution


Here you describe how you work out the (logarithm of the) expression to somethng that you can integrate

Work carefully and check each small step -- also check if what you wrote is what you meant to write

I suppose that in your case you want to follow a worked out example in the book, but even there the same rules apply.
That's why I remarked sourly that what's on the second line (after the first = sign) is not what is at the end of the first line.
 
Question: A clock's minute hand has length 4 and its hour hand has length 3. What is the distance between the tips at the moment when it is increasing most rapidly?(Putnam Exam Question) Answer: Making assumption that both the hands moves at constant angular velocities, the answer is ## \sqrt{7} .## But don't you think this assumption is somewhat doubtful and wrong?

Similar threads

  • · Replies 6 ·
Replies
6
Views
2K
  • · Replies 3 ·
Replies
3
Views
2K
  • · Replies 4 ·
Replies
4
Views
2K
  • · Replies 14 ·
Replies
14
Views
2K
  • · Replies 2 ·
Replies
2
Views
2K
  • · Replies 3 ·
Replies
3
Views
2K
  • · Replies 8 ·
Replies
8
Views
2K
Replies
11
Views
2K
  • · Replies 5 ·
Replies
5
Views
2K
  • · Replies 2 ·
Replies
2
Views
1K